« first day (1590 days earlier)      last day (3433 days later) » 

7:00 PM
@skullpatrol Oh, OK. Just asking :)
@robjohn OK, I wouldn't do that :)
 
@robjohn Everyone can vote?
 
hi all
 
@MarcGato Si j'essaie dans le sens direct, j'obtient : soit $x\in E,$, il existe $\epsilon>0$ tq $B_o(x,\epsilon)\in A$. On veux $\forall\varpi>0,\exists a,\forall w\in B_o(x,a),f(w)\in B_o(f(x),\varpi)$ mais je suis bloqué la
 
@Hippalectryon Tu vois le latex sur le chat?
 
@MarcGato Oui
 
7:06 PM
comment? perso je suis obligé de copié collé
sur poser une question à chaque fois
 
@robjohn One more question, the policy site says that the minimum age for the user is 13, but how if there's a user who signed up this site when he was under 13 and now he is above 13?
 
@MarcGato Je reformule ce que j'ai écrit plus haut :On veux $\forall\varpi>0,\exists a,\forall w\in B_o(x,a),f(w)\in B_o(f(x),\varpi)$. J'ai : soit $x\in E,$, on pose $B=B_o(f(x),\epsilon),A=f^{-1}(B)$, alors $B_o(x,\epsilon)\in A$.
 
@robjohn What will happen to this user? Will his account get suspended?
 
@MarcGato I think that this page is still mostly accurate.
@Anastasiya-Romanova秀 I don't know about that. You'd need to ask on meta and see if a community manager can answer that.
 
7:10 PM
@Hippalectryon je clique juste sur start ChatJax ? Bref sinon tu as $f^{-1}(B)$ qui ouvert et contient x donc contient..
 
@MarcGato Mets Start Chatjax dans tes bookmarks, et clique dessus lorsque tu es sur cette page
 
ah bah oui tu l'as écrit à la fin donc c'est bon
 
Did I just enter France?
 
@MarcGato Mais je ne vois pas comment continuer
 
@MarcGato So you need at least 150 reputation to vote. Coming from another site would automatically give you 100 reputation here (assuming you have 100 reputation there), so you'd still need to do some work here to be able to vote.
 
7:12 PM
LOL @Studentmath
 
@Studentmath Not yet, you still have to pay the entry tax
 
bah c'est équivalent à la continuité.
 
Who would pay to enter France?
 
@Hippalectryon and @MarkGato are planning to blow up New York by dropping nukes.
 
@MarcGato Je ne vois pas l'équivalence..
 
7:14 PM
tu utilises les propriétés ensemblistes tel que A inclus dans B donne f(A) inclus dans f(B) pareil pour la réciproque.
 
Cool
 
@BalarkaSen SHHH don't tell
 
Oh, I had a quick question before french ate up my brain -
 
@MarcGato Oh mais bien sûr ! Merci
 
But I think I am doing it right so nevermind.
 
7:15 PM
you always do that @Studentmath
 
@robjohn Oh OK, thank you for answering my question :)
 
@Hippalectryon Désolé je suis un débutant c'est quoi mes bookmarks?
 
Ask quick questions? @Balarka
 
stating that you want to ask a quick question and then changing your mind
:P
 
@MarcGato Favoris. Tu utilises quel navigateur ?
 
7:15 PM
Sorry guys I am terribly long to think in English..
 
@MarcGato personally I don't mind, seeing I am the only one commenting on it might be good to say that
 
Being "terribly long" has nothing to do with thinking in English ... :P
 
ah d'acc, je vais essayer merci @Hippalectryon
 
@MarcGato is saying that he'd mercilessly kill every New Yorker.
 
@BalarkaSen or equivalently I am better to think in French :)
@robjohn Thank you! I have more than 150 so no problem..
 
7:18 PM
@BalarkaSen Relevant
 
@MarcGato Your bookmarks are like this or this.
 
Hi there i am kinda stuck with an linear algebra question is there someone who migth help me?
 
@Balarka Okay, I will ask! So I have the Metric space $X$ over the interval $[0,2]$, and the distance metric $d(x,y)=|\lfloor x \rfloor +x - y -\lfloor y \rfloor |$. I have $f:X\to \Bbb R$, $f(x)=x, x\le 0.5; f(x)=x+1, x>0.5$. I want to show discontinuty, so is the following reasoning correct:
 
@KellyBlunie What's the question ?
 
@Kelly try us
 
7:25 PM
58 mins ago, by robjohn
@KellyBlunie just ask the question. If there is someone who can answer, they will
 
@Studentmath Such a stupid metric.
 
I will show discontinuty at $0.5$. If I take $\epsilon=1/4$ for example, then $D((f(0.5),\epsilon))=(0.25,0.75)$
And yet $f(D(0,5,\lambda)$ will include $0.5+\lambda /2$ for any $\lambda$. Thus I get discontinuty.
@Balarka well it's just for the example, to see I get the concepts right
 
@robjohn Yes, I just have to add math.ucla.edu/~robjohn/math/mathjax.html to my bookmark and run it when I am here ?
because I cannot add start ChatJax to my bookmark
 
@KellyBlunie ask away :-)
 
Anyhow, that reasoning is correct, right? I am not overly-confused/silly
 
7:29 PM
@MarcGato Pourquoi n'arrves-tu pas à l'ajouter à tes bookmarks ?
@MarcGato Si tu es sous Chrome tu peux simplement drag'n'drop
 
@Hippalectryon bha je clique droit sur start ChatJax je peux juste copier le lien ou l'ajouter à itunes
 
Sorry for the delay i was editing the question the question is in here math.stackexchange.com/questions/1063284/…
 
You're confusing me @Studentmath. What is $D$?
 
iTunes ? Ah tu es sur Mac. Tu as Chrome ou Safari ? Tu es sur ordi ou portable ? @MarcGato
 
Sorry, open ball $B(x,r)$
We mark it at times by $D(x,r)$
 
7:32 PM
portable macbook safari
 
There is no play of open balls/disks in continuity.
 
@MarcGato Ta barre de bookmark est affichée en permanence ?
 
If it it is confuse my question let me know
 
@MarcGato No, you install the bookmarks from that page. The "start ChatJax" bookmark is the one to use for chat
 
@Hippalectryon comment? la barre des Signets est toujours visible si je mets mon curseur vers le haut.
 
7:33 PM
@MarcGato Essaie de drag-n-drop the lien dans cette barre
 
I already have one answer but dont actually answer my question or i understand it
 
@KellyBlunie "If every linear combination of A produces V, then Span(A)=V" that is false
@KellyBlunie Or at least not said well
 
@Studentmath Wait why is $B(f(1/2), 1/4)$ what you wrote?
 
@Hippalectryon marche pas ^^
@robjohn it doesn't work on my laptop
 
@KellyBlunie Maybe you meant "If the reunion of all the possible linear combinations of $A$ produces $V$, then ..." ?
 
7:35 PM
@Balarka Actually, we know $f$ is continious in $x_0$ iff for every $\epsilon$ there is some $\delta$ so that $f(B(x_0,\delta ))\subseteq B(f(x_0),\epsilon)$
 
That the argument that is on test
 
@Studentmath Weird definition.
One usually makes the epsilon delta definition with simple metric stuff
 
@MarcGato Do you have the "start ChatJax" bookmark in your bookmark bar?
 
Not the definition, but it's equivalent. I've shown that in some previous exerciss.
 
@BalarkaSen What definition are you using?
 
7:35 PM
But that works, yeah
 
Well yeah I could actually do the same with that.
 
@robjohn No, he's not able to put it here
@robjohn He's on a portable macbook with safari
 
@MarcGato The standard definition.
But I'm not using anything at the moment.
 
@MarcGato Va dans View->Show bookmark bar
 
OK, that looks good @Studentmath. I haven't checked your calculations of open balls.
 
7:37 PM
@MarcGato Et réessaye le drag-n-drop
 
@BalarkaSen Sure, sometimes some definition are more interesting than the standard one.
 
@Balarka cheers :)
 
@MarcGato Yes, I know. I was helping @Studentmath and was wondering what he was using.
 
@Hippalectryon i add a only
To my post
 
@KellyBlunie That's not what I meant. As you write it, "every linear combination of A produces V" means that any linear comb of $A$ makes $V$
 
7:39 PM
@MarcGato can you add it as a bookmark? right click on the link and there should be a pop-up menu that allows you to add a bookmark
 
@Hippalectryon It works!!!
 
Hello!!!

Let $\mathbb{F}_{p^m}$ and $\mathbb{F}_{p^n}$ be the subfields of $\overline{\mathbb{Z}}_p$ with $p^n$ and $p^m$ elements respectively.

How can I find the field $\mathbb{F}_{p^m} \cap \mathbb{F}_{p^n}$ ??
 
@Hippalectryon yes only if it is true
 
@Hippalectryon there is a bookmark bar in Safari
 
7:40 PM
@robjohn No I could not do this, but @Hippalectryon find the solution.
Happy end.
 
@Hippalectryon but could be not
 
@BalarkaSen OK :)
 
@KellyBlunie What I mean is that a linear combination of $0$ elements is a linear combination of $A$, and obviously is not $V$.
 
@Studentmath I've got a problem for you.
 
@Balarka shoot
If it's not too harsh group-theory, I might just solve it
 
7:45 PM
@Hippalectryon i don`t follow you,
 
@Hippalectryon I'm on a portable MacBook, but using FIrefox. I didn't think the bookmark bars were that different.
 
@KellyBlunie Saying "every linear combination of A produces V" is the same as saying "any linear combination of A produces V". However, a linear combination of $0$ elements of $A$ does not produce $V$. Contradiction.
 
Does anyone know some (as basic as possible a) paper or article/blog post with uses of the Lefschetz number? I've seen some examples already but it would be nice to see it 'in the wild'.
 
@robjohn I didn't really know myself either, I use Chrome.
 
@Hippalectryon My real question is with the determinant being zero and what that implies
 
7:48 PM
@Studentmath I was actually going to give you group theory.
Sorry, the internet is a bit annoying in this end.
 
@KellyBlunie You wrote " lets say the set A spans a vector space V only If every linear combination of A produces V, then Span(A)=V", i'm just telling you that it's false
 
@Balarka I might manage, depends on the toughness - but try me
 
Quick question, solving an IVP where: $y(2)=0$ And the solved implicit form is: $y(1+y)=x^2+C$ How do I solve it for IVP now?
 
Hey guys, is it true that Hoffman and Kunze Linear algebra has had 3rd edition ?
I found a page for 3rd edition in Amazon, but it seems not really existed
 
@Studentmath $G$ be a a group with subgroups $H$ and $K$. Under the action of $H \times K$ on $G$ via $(h, k) \cdot g \mapsto hgk^{-1}$, the orbit of some element $g$ of $G$ is $HgK = \{hgk : (h, k) \in H \times K\}$. This is called an $(H, K)$-double coset of $g$ in $G$. The set of all double coset is denoted $H\setminus G /K$. Prove that $|H \setminus G /K| = |K| \cdot |H \cap gKg^{-1}|$
OK, I gotta go. The internet connection here is a bit troubling.
 
8:01 PM
@Hippalectryon what is wrong? And besides that being wrong ,ignoring it, can you say if the determinant is zero why a general vector from V is not contain in A
 
Typo : I mean $|K| \cdot |H : H \cap gKg^{-1}|$
 
@balarka reading
 
@KellyBlunie The determinant has nothing to do with the existence of a solution.
 
My book states:there is at least one choice of u for which this system will not have a solution and so u can not be written as a linear combination of these vectors. And that there are in fact infinitely many choices of u that will not yield solutions.
 
@KellyBlunie For instance take the matrix $\begin{bmatrix}1&0\\1&0\end{bmatrix}$ for a given $u$
 
8:06 PM
If you want i can give you the exercice can be more clear
 
That would be better indeed
 
@Balarka that's really nice, think I might manage with enough time
 
It's not entirely trivial @Studentmath. A bit nonobvious, in my opinion.
Anyways, have fun with it.
 
Thanks. $G$ is finite, right?
 
Yup.
There's a bit trick involved in the problem. As a hint, fiddle with the actions :)
 
8:11 PM
Determine if the following sets of vectors will span R3 . v1=(1,2,-1) ,v2=(3,-1,1),v3=(-3,8,-5)
 
Yikes, @Studentmath. I mean $|HgK| = |K|\cdot|H : H \cap gKg^{-1}|$
Computing $|H \setminus G / K|$ is a different beast, although would be nice to think about.
 
Oki
 
@KellyBlunie You can look at the determinant of $\begin{bmatrix}1&2&-1\\3&-1&1\\-3&8&-5\end{bmatrix}$
 
And then determine the coeficient matrix then compute det(B)=0 then say
 
@KellyBlunie Do note that the matrix I wrote above it not the one you were referring to in your MSE question, though
 
8:15 PM
Is there any difference between matrices with square parentheses and curved parentheses, @Hippa?
 
@Khallil None that I know of
I'm just used to bmatrix
 
@skullpatrol Good luck against the Chiefs Sunday :)
 
Ah, as opposed to pmatrix.
 
@Balarka Can I define something like $h(gK)=hgK$ on the left cosets of $K$? it fits with everything so I don't see why not
 
@KellyBlunie The coefficient matrix of ???? what is B ?????
 
8:19 PM
Nah don't think that's gonna help.
 
@Hippalectryon the matrix you have it is the transpose i have you formed a linear combination of vectors and then forme a system of equations based on the components then forme the matrix and that is the transpose of your matrix lets call this transpose B however the determinant is zero in either
there is at least one choice of u for which this system will not have a solution and so u can not be written as a linear combination of these vectors. And that there are in fact infinitely many choices of u that will not yield solutions.
 
Then it does not span $R^3$
 
And them state this
Yes it does not
Why the book say that statement
there is at least one choice of u for which this system will not have a solution and so u can not be written as a linear combination of these vectors. And that there are in fact infinitely many choices of u that will not yield solutions.
After compute determinante
 
You don't even need that statement to get the result
 
Yes indeed
 
8:25 PM
A "coefficient matrix" of a base is of course invertible
Which directly implies that if the det is $0$, your vectors don't form a base
And hence don't span V
 
But i was reading a subject further ahead basis and dimension and i realize i didint know something and i got back
 
Hi guys, I know that $\ell_P^*$ is isometrically isomorphic to $\ell_q$. But I am having a hard time understanding this. An element in $\ell_p^*$ is a function from $\ell_p \rightarrow \mathbb{R}$ .. so how a function be the same as an element of $\ell_q$ (which are sequences).
 
@Hippalectryon my question is why we conclued that there is at least one vector that doesnt belong to $V$ because det(B)=0 B is the coefficient matrix
 
@KellyBlunie Det B=0 => the vectors don't span the space => there is a vector that doesn't belong to your span
If all vectors were belonging to the span, then by definition the vectors would span the space
 
Yes but you do the first impliction
How do you prove it
Or how do explain it
Correction: how do you do the 1st implication
 
8:38 PM
I already told you
The matrix of a base needs to be invertible
Or, do you need to prove that too ?
 
Ok i need to read that sectiin first
@Hippalectryon f a matrix in not invertible singular,it is true that could have no solution or infinitily many solutions? In case of a square matrix
Correction: if
 
@KellyBlunie I don't really understand that sentence ...
 
If a a matrix is singular then the set of solution is no solution or infinitily many solotions
Solutions
 
@KellyBlunie The set of solutions... to what ? A matrix isn't a set of equations ...
 
Because it it is non-singular then has to have exactly one solution to the system Ax=b
Yes of course
If Ax=b if detA is singular then has no solution or infinitly many solution
DetA=0
 
8:48 PM
Indeed
 
The oposite is non-singular one solution to the system Ax=b
 
Ok thank you for your help,time and patient with me
 
No problem :)
 
See you
 
8:55 PM
@Balarka think I got it, not sure if I did it right though
I computed the orbital of $|HgK|$ and then showed the equality via that
 
How did you do it?
Your idea above was on the right track though. You can thinking about the double coset actions as $H$-action on $G/K$ instead.
Whoops, I can't stay. Post your proof and ping me, I'll check later.
And somebody ping @Mike and tell me that it's been a week.
 
Okay, so I actually went with how H acts on $gK$, and I defined it as I said: $h(gK)=hgK$. So I wanna know the size of the orbit of $gK$, it is naturally the size of $hgK| h\in H$. I will mark it by $A$. Now we want to compute the order $HgK$. It is the union of $A$ disjoint sets of the form $hgK$, each of order $|K|$ so $A*|K|$ @Balarka
Next I want to compute $H:H\cap gKg^{-1}$ (the order thereof), and show it is equal to $A$.
 
Hello!!! Can someone help me at an exercise of field theory??
 
9:12 PM
But then a stabilizer $h\in H$ upholds $hgK=gK$ which leads me to $h\in gKg^{-1}$
Erm.. no wait, that's fine.
Yeah, and then I know that the the stabalizer is $H\cap gKg^{-1}$, and since the orbit of $gK$ is the index of the stabilizer, we complete the proof. I think.
 
practical question: I'd like to see an English translation of a certain arXiv paper which is in Russian i.e. Cyrillic. At this point I really just want to figure out if there's anything in that version which isn't in the English language papers by the same author, so I don't need high quality. Are there any obvious tools?
 
r9m
@Semiclassical copy paste text to google translate ?! =P
 
that would work, if I could figure out how to copy-paste cyrillic :P
 
@MikeMiller @DanielFischer Let $\mathbb{F}_{p^m}$ and $\mathbb{F}_{p^n}$ be the subfields of $\overline{\mathbb{Z}}_p$ with $p^n$ and $p^m$ elements respectively.

How can I find the field $\mathbb{F}_{p^m} \cap \mathbb{F}_{p^n}$ ??
 
r9m
@Semiclassical you can't select and copy the text form the pdf ?
 
9:19 PM
Doesn't seem to work: copy-paste doesn't know how to interpret Cyrillic characters
For reference, its ID is 0505015 in nlin
 
r9m
umm .. installing Cyrillic package in the pdf viewer you are using sounds overkill ... I have no other idea atm .. :o
 
@MaryStar The nonzero elements of $\mathbb{F}_{p^k}$ are the elements of $\overline{\mathbb{Z}}_p$ with $x^{p^k-1} = 1$. So for $x\neq 0$, we have $$x\in \mathbb{F}_{p^n} \cap \mathbb{F}_{p^m} \iff x^{p^n-1} = x^{p^m-1} = 1.$$ Suppose $n > m$. It then follows that $x^{p^n - p^m} = \bigl(x^{p^{n-m}-1}\bigr)^{p^m} = 1$. Since the Frobenius homomorphism is injective, it follows that $x^{p^{n-m}-1} = 1$, i.e. $x\in \mathbb{F}_{p^{n-m}}$.
Iterate the reasoning to find the $k$ such that $\mathbb{F}_{p^n} \cap \mathbb{F}_{p^m} = \mathbb{F}_{p^k}$.
 
@KellyBlunie You can perform reversible column operations that don't change the determinant that diagonalize any matrix $M$. The concatenation of those column operations can be represented by an invertible matrix on the right (that doesn't change the determinant). So suppose that $MA$ is diagonal and $A$ does not change the determinant of $M$. Since $\det(MA)=\det(M)=0$, one of the diagonal elements of $MA$ must be $0$. Choose a vector $v$ with a $1$ at that position and $0$s elsewhere.
@KellyBlunie consider $Av$. Since $A$ is invertible $Av$ cannot be $0$. However, $MAv=0$
 
@Chris'ssis Sorry, I was slept. I can't see your chat properly as I'm online via my tab. Maybe later...
Anyway, have you all ever heard about Libya Algebra? :D
 
@DanielFischer I understand... But, what do you mean by "Iterate the reasoning to find the $k$ such that $\mathbb{F}_{p^n} \cap \mathbb{F}_{p^m} = \mathbb{F}_{p^k}$." Isn't $k$ equal to $n-m$ ??
 
9:36 PM
@MaryStar Generally, no. We have $$\mathbb{F}_{p^n} \cap \mathbb{F}_{p^m} \subset \mathbb{F}_{p^{n-m}},$$ but generally, not equality. Note that $$\mathbb{F}_{p^n} \cap \mathbb{F}_{p^m} \subset \mathbb{F}_{p^{n-m}} \cap \mathbb{F}_{p^m}.$$
Well, in the last, we do have equality, $$\mathbb{F}_{p^n} \cap \mathbb{F}_{p^m} = \mathbb{F}_{p^{n-m}}\cap \mathbb{F}_{p^m}.$$
 
@Venus The idea is to compute it without pen and paper, without using the asymptotic expansion of the harmonic number $$\lim_{n\to\infty} n\left(H_n-\log(n)-\gamma\right)$$
 
@DanielFischer How can we find then $k$??

And why do we have in the last equality??
 
@Chris'ssis I use my tab & your LaTeX can't be rendered on my screen
 
@MaryStar We have a function, call it $f$, with $f(n,m) = f(n-m,m)$ if $n > m$, $f(m,m) = m$, and $f(n,m) = f(m,n)$, and $f(m,1) = 1$. What could $f$ be?
 
It seems I have seen your limit in Wiki about digamma function or Euler's constant. Let me check it later.
Right now I LOL like crazy :D
 
r9m
9:45 PM
@Venus ? why ?
 
@r9m Have you ever heard about Libya Algebra?
 
r9m
@Venus what is that ? :o
 
hello
 
Algebra from the country Libya, @r9m and @Venus?
 
See the comment below Arkamis' official nomination. I can't send you the link
 
9:48 PM
Compute without pen and paper (without using the asymptotic expansion of the harmonic number) $$\lim_{n\to\infty} n(2H_n-H_{\large n^2}-\gamma)$$
@Venus ^^^
 
That's the proof that you can't rely too much on autocorrect in your phone :D
 
I don't see the appeal of computing things without pen and paper.
If you have a piece of paper and a pen, why not use them?
Each to their own, I guess.
 
anyone want to help with a couple algebraic geometry questions?
 
@daOnlyBG What is your question?
 
It's right here- a user was kind enough to write a response, but with no follow-up, so I'm a bit uncertain about a few matters
1
Q: Proving $V(f)$ is finite when $f$ is non-constant

daOnlyBGMy problem asks me to show that if $f$ is non-constant, then $\mathbf{V}(f)$ is finite. Assume that $f \in \mathbb{C}[x]$. If $f$ was an ideal, this would be straightforward; however, $f$ is merely a polynomial described above. I suppose by "non-constant" the prompt means "a polynomial with max...

 
9:53 PM
@Chris'ssis How come you're so mean to me? I've just woken up yet you give me very difficult question. I can't even the first one tho :D
 
@Venus :D:D:D Some suggest that I'm mean in general, but I'm actually very kind all the time! :-)
@Venus Take your time! I'm pretty good at this kind of questions, I have much practice in this area.
 
@DanielFischer How could we find such a $f$ ??
 
IIRC this thread was one of the weirdest ever on math.SE in that all answers, and they were plenty, were downvoted. One of them even got a bounty, which is verified in the comments.
114
Q: Evaluate $ \int_{0}^{\frac{\pi}{2}}\frac{1}{(1+x^2)(1+\tan x)}\,\mathrm dx$

juantheronEvaluate the following integral$$\int_{0}^{\Large\frac{\pi}{2}}\frac{1}{(1+x^2)(1+\tan x)}\,\mathrm dx$$ My Attempt: Let $$\tag1 I = \int_{0}^{\Large\frac{\pi}{2}}\frac{1}{(1+x^2)(1+\tan x)}\,\mathrm dx.\\[15pt]$$ Putting $x=\frac{\pi}{2}-x$ and using property $\displaystyle\int_{0}^{a}f(x)...

What happened to all the answers?
Any 10k user out there that can be of assistance? Were they voted to delete or something?
 
r9m
@DanielR They Burnt in HeLLFire !!!!
 
:D
 
9:56 PM
@MaryStar You should be familiar with such a function. It remains to see that the function is unique after you recognised it.
 
@Chris'ssis No!! You're killing us softly with your difficult question. I even once had a nightmare with your integral problem :P
 
@Venus :D:D:D
 
r9m
@Venus tell me about it ! me too :( X(
 
@Venus By the way, did you see this one I posted today? $$\int_0^1 \space _3F_3(1,1,1;2,2,2;\log(x)) \ dx=\frac{3}{4}\zeta(3)$$
 
r9m
@Chris'ssis what does that $_{3}F_{3}(..;\log x)$ mean ?
 
9:59 PM
@evinda what do you think?
 
@DanielR Fourteen answers deleted by owner, thirteen by moderators or 20k users.
 
@r9m Hi five then! We should ban @Chris'ssis from this chat room for good. She tortures us with his problems :D
 
@r9m This you need to discover on your own :-)
 
*her
 
@Venus No! :-) (although some would be pretty happy if this happened)
 
10:00 PM
@DanielFischer Thanks! There must have been a deletion spree going on wiping out all om them at once, or at least in a quite short span of time. Strange...
 
@daOnlyBG I haven't looked at it yet.. I will take a look at it later and I'll tell you if I have an idea..
 
ah ok, thanks
anyone else?
here's my conundrum: I have to show that if f is non-constant, then V(f) is finite
 
No, I've not seen yet. The li one in Jack's answer makes me confused. I totally don't get it his answer.
 
r9m
@Venus I'm against banning (or banishing) anyone ! :P (especially not her .. her posts are more than half the reason I stuck to this chat for so long .. torture is just a light price you pay .. nothing severe :P)
2
 
where f is a polynomial in C[x]
 
r9m
10:02 PM
@DanielFischer my fingers are itching ! (evil grin) ;) (but I won't do anything .. no worries)
 
@DanielR No, actually the answers were deleted fairly well-spread from April '13 until recently.
@r9m If you know a good hint, without giving everything away ...
 
OK guys, I wanna continue my sleep as the adzan from the mosque has already stopped. See you later... :-)
 
Night, @Venus! ^_^
 
@DanielFischer I got stuck right now... I cannot think of such a function... Could you give me a hint??
 
@DanielFischer Oh, I see.
 
10:05 PM
@Khallil it should be good morning :D
 
@MaryStar Try working it out for some small $m,n$.
 
r9m
@DanielFischer I could say His name .. :P
 
Haha! You're such a rebel, @Venus!
 
@r9m You're just too kind to @Chris'ssis :P
 
@r9m Thrice, to summon him?
 
10:06 PM
@r9m thanks, thanks, thanks ... :D
@Venus :D
 
r9m
@DanielFischer haha ;) LOL
@Venus kind ?! not exactly !! she's just adorable ! ;) that's all =P
2
@Chris'ssis :D
 
@r9m Nicely emphasized! :D
I have a challenge for you all, the limit I received yesterday from @robjohn
 
$\mathbb{R}[i]$ is just $\mathbb{C}$, right?
 
Can we compute this one in an easy way (like one line)? $$\lim_{a\to\infty}a^4\int_0^\infty\frac{\cos(ax)}{1+x^3}\mathrm{d}x$$
 
@Chris'ssis the first or the second? I see, the first.
 
10:15 PM
@Khallil The first one.
 
r9m
@Chris'ssis $-6$ ?
 
@r9m Kind of. :-) How you did it? :-)
 
r9m
@Chris'ssis I was working on the same problem @robjohn was (on the main) and then I read his answer ! =P
 
@r9m Ah, OK. :-)
 
14 mins ago, by Khallil
$\mathbb{R}[i]$ is just $\mathbb{C}$, right?
^_^
 
10:28 PM
yup
 
Thanks, @Mike!
 
r9m
@DanielFischer sama is what I did here correct ?! :o .. also darij says $B = p\left(B^m\right)$ .. I don't understand how I could prove it ! :o
 
@DanielFischer Unfortunately I have no idea...
 
'Sama', @r9m.
 
Guys real quick, if f is a group homomorphism and bijective, how do I show f^{-1} is a homomorphism (I hate proofs that fly close to definitions), I just applied f to both sides of f(a)f(b)=f(ab) where means inverse, that's okay right? [quick yes/no, I'm not stuck]
 
r9m
10:35 PM
@MaryStar Euclidean algorithm ! :-)
@Khallil ?! what's wrong with 'sama' ? :O
 
@r9m How can we use the Euclidean algorithm ??
 
Given $n,m \in \mathbb{N}$, $n$ divides $m$ with remainder $r$ if $\exists q \in \mathbb{N}$ such that $m=qn + r$ where $0 \leq r < n$, right @r9m?
 
r9m
@MaryStar it is the euclidean algorithm (the function is gcd function)
 
I'm sorry, it's just that I hardly ever see people using honorifics on here @r9m!
 
r9m
@Khallil right ! ..and just divides (without mention of remainder) means $r = 0$
 
10:39 PM
Sorry! I just realised what I wrote was total nonsense. I've edited it now. Does it look right @r9m?
Yep. Ah, the editing time has passed, but you're saying that I should really have written that $r \in (0, n)$ where $r \in \mathbb{N}$, right @r9m?
 
r9m
@Khallil you wrote that .. ! where 0≤r<n,
 
@r9m You're a bit short on why $d_i^mw_{ij} = w_{ij}d_j^m$ for all $i,j$ implies $d_i w_{ij} = w_{ij}d_j$, but it's correct. To see that you can write $B = p(B^m)$ for some polynomial $p$, first diagonalize, then you need a polynomial with $p(d_i^m) = d_i$ for $1\leqslant i \leqslant n$. Lagrange knew that such a polynomial exists.
 
r9m
@DanielFischer ah !! now it makes sense !! Thanks ! :-)
 
Hmmm. I don't see what you're getting at @r9m. :/
 
r9m
@Khallil you wrote it right ! no worries !
 
10:44 PM
So this is right, @r9m?
6 mins ago, by Khallil
Given $n,m \in \mathbb{N}$, $n$ divides $m$ with remainder $r$ if $\exists q \in \mathbb{N}$ such that $m=qn + r$ where $0 \leq r < n$, right @r9m?
 
r9m
@Khallil absolutely !
 
Oh yea. Of course it is. The remainder would just be $r=0$ if $n \mid m$ straight off.
The Well Ordering Principle just states that any subset of $\mathbb{N}$ has a least element right, @r9m?
 
r9m
@Khallil right !
 
@Khallil non-empty
 
(Sorry if these seem like random questions. These are all part of one module I'm doing that I'm kinda struggling with!)
Of course! facepalm It's crucial details like that, that I keep forgetting! Thank you, @Daniel!
 
10:47 PM
@r9m Ok!! Thank you!!! :-)
 
r9m
irk .. I missed that too ! MoG ! (o_o)
@MaryStar ;) Wc !
 
@DanielFischer Knowing that the function is the gcd function, what do we conclude??
 
What's the $\gcd$ function, @Mary?
 
r9m
@Khallil greatest common divisor function .. gcd(m,n) is gcd of m and n ..
 
Oh! It's the function that returns the greatest number that divides both the numbers put in.
I've only seen it as $\text{hcf}$, the highest common factor. Same thing though!
^_^
 
10:53 PM
@MaryStar $$\mathbb{F}_{p^n} \cap \mathbb{F}_{p^m} = \mathbb{F}_{p^{??}}$$
 
r9m
@Khallil aha ! you are studying at GB right ? makes sense !
 
Yep! We use all sorts of non-universal lingo, @r9m. =P
See! It's more $\mathbb{N}$ for us to say lingo instead of jargon. Haha!
 
r9m
@Khallil I was introduced to gcd as hcf too in my primary school :-)
 

« first day (1590 days earlier)      last day (3433 days later) »